express 96 as the product of prime factors
need it asap
plzz write the method too​

Answers

Answer 1

Given:

The given number is:

[tex]96[/tex]

To find:

The product of prime factors of given number.

Solution:

We have,

[tex]96[/tex]

The prime factors of given number are:

2 | 96

2 | 48

2 | 24

2 | 12

2 | 6

3 | 3

  | 1    

So, 96 can be written as:

[tex]96=2\times 2\times 2\times 2\times 2\times 3[/tex]

Therefore, the 96 as the product of prime factors is [tex]96=2\times 2\times 2\times 2\times 2\times 3[/tex].


Related Questions

Please help me the picture is above I’ll mark as brainliest. I really need the answer

Answers

I think that is is 65 cubic inches in a box

the pythagorean theorem equation

Answers

A^2+b^2=c^2
Is the Pythagorean theorem

Answer:

(Altitude)^2+(Base)^2=(Hypotenuse)^2

I need the answers and it's due today, please help

Answers

Answer:

1. 3

2. 1

3. 2

4. 4

5. 5

Step-by-step explanation:

A petrol can is a rectangular prism with base measurements 15 cm by 30 cm. If the can has capacity 18 liters, find its height.​

Answers

Answer:

40

Step-by-step explanation:

The first step is to calculate the base area of the prism

= 15×30

= 450

The volume is then calculated as follows

= 450×h

= 450h

The capacity is 18 liters

= 18×1000

= 18,000

Therefore the height of the rectangular prism can be calculated as follows

450h= 18,000

h= 18,000/450

= 40

Hence the height is 40 cm

A 2km long stretch of road needs to be resurfaced. The width of the road is 8.3m. Model this stretch of road as a rectangle.Work out the area of road that needs to be resurfaced in metre squared. Resurfacing this road costs £30 per square metre What is the total cost to resurface this road?

Answers

1 km = 1000 meters

2 km = 2 x 1000 = 2,000 meters

Area = length x width

Area = 2,000 x 8.3 = 16,600 square meters

Cost:

Multiply total area by cost per square meter:

16,600 x 30 = £498,000

The two triangles are similar. What is the value of x? Enter your answer in the box. x = Two right triangles, one smaller than the other, back to back, so that their right angles form a straight angle. The two acute angles along the straight angle are congruent to each other. The overlapping part of the legs is labeled 12. The part of the overlapping side that extends above the smaller triangle is labeled 3. The leg of the smaller triangle that is a ray of the straight angle is labeled 3 x plus 1. The leg of the larger angle that is a ray of the straight angle is labeled 4 x.

Answers

Answer:

The two triangles are similar.

What is the value of x?

Enter your answer in the box.

x=

The value of x is 7 units.

What is Similarity of Triangles?

Two triangles are said to be comparable if their two sides are in the same ratio as the two sides of another triangle and their two sides' angles inscribed in both triangles are equal.

Given :

AD = 6 units , BD = 8 units, m∠ABC = m∠DBE = 90° and              

∠DEB ≅ ∠ACB

Now, AB = AD + BD

              = 8 + 6 = 14 units

Now, In ΔABC and ΔDBE,

∠DBE = ∠ABC ( Each of 90° )

∠DEB = ∠ACB ( given )

So, By using AA postulate of similarity of triangles , ΔABC ~ ΔDBE

Now, proportion of the corresponding sides will be equal.

AD/ DB= BC/ BE

14/8 = 3x/ 2x-2

4x= 28

x= 7

Hence, the value of x is 7 units.

Learn more about similarity of Triangle here:

https://brainly.com/question/11923416

#SPJ6

Let $z$ and $w$ be complex numbers satisfying $|z| = 5, |w| = 2,$ and $z\overline{w} = 6+8i.$ Then enter in the numbers \[|z+w|^2, |zw|^2, |z-w|^2, \left| \dfrac{z}{w} \right|^2 \]below, in the order listed above. If any of these cannot be uniquely determined from the information given, enter in a question mark. Don't even know where to start.

I got 49 for the first one, 100 for the second one, 9 for the 3rd one, and 6.25 for the 4th one. But it was wrong, so I don't know how to do this question.

Answers

Answer:

1st entry: 41

3rd entry: 17

Step-by-step explanation:

The 1st and 3rd entry are incorrect.

We will use the following for the 1st and 3rd:

[tex]|z+w|=|z|^2+|w|^2+2R(z\overline{w})[/tex]

[tex]|z-w|=|z|^2+|w|^2-2R(z\overline{w})[/tex]

where [tex]R(z\overline{w})[/tex] means 'real part of [tex]z\overline{w}[/tex]'.

Let's do part 1 now)

25+4+2(6)

29+12

41

Let's do part 3 now)

25+4-2(6)

29-12

17

Use the figure below, If m

Answers

Answer:

m anglw ABD answers is 50 degree

the answer is 50 degrees

A newsletter publisher believes that 30% of their readers own a Rolls Royce. A testing firm believes this is inaccurate and performs a test to dispute the publisher's claim. After performing a test at the 0.02 level of significance, the testing firm fails to reject the null hypothesis. What is the conclusion regarding the publisher's claim

Answers

Answer:

The evidence is not sufficient at the 0.02 level of significance to reject the claim that the percentage is 30%.

Step-by-step explanation:

Let's first state the null and alternative hypotheses for this research;

H0: p = 0.30

Ha: p ≠ 0.30

Now, since the bull hypothesis is the claim and he fails to reject the null hypothesis from the research, we will conclude that;

The evidence is not sufficient at the 0.02 level of significance to reject the claim that the percentage is 30%.

What is the Order of Magnitude of the number of pennies equivalent to a quarter?​

Answers

Answer:

It is mainly used while doing scientific notation. If two numbers differ by one order of magnitude, one is about ten times larger than the other. If they differ by two orders of magnitude, they differ by a factor of about 100.A quarter is 0.25 dollars so the ratio between 1 dollar and a quarter is 1/0.25(it means the dollar and the quarter which is 0.25 dollars) and that equals =4

Help please I’ll give you the brainliest I really got to get this done

Answers

Answer:

56

Step-by-step explanation:

Write an equation of the parabola that passes through the point (46, - 150) and has x-intercepts - 4 and 56 . Then find the average rate of change from x= -4 to x=6.

An equation is y =

The average rate of change is

Answers

Answer:

6w83ywytahausjsshshfwgwyee

Fbjdbdbdhhdhdbddbdbbdbddbbd so yea

What is the slope-intercept form of the equation representing this function?

Answers

Answer:

y = 3x - 7

Step-by-step explanation:

The line rises 3 in y for every run of 1 in x

slope = 3

The y-intercept

is -7

Therefor

y = 3x - 7

Y=3x-7 the 3 is the slope

Which expression is equivalent to this quotient?

[tex]\frac{1}{x+5} \\\frac{x+3}{5x+15}[/tex]

Answers

I think this was your problem.. hope this helps!

Will give branliest
Find l.
l= [?] in

Answers

L= Square Root of (h^2 + r^2)
h=15 and r= d/2= 12/2= 6
So, (15^2+6^2)=
225+36= 261
L= Square Root of [261] in
L= the square root of 261 in

what is the total cost for an item that costs $140.00 and has a sales tax rate of 7.5%

Answers

Answer:

$150.5

Step-by-step explanation:

CP of an item = $140

sales tax rate = 7.5%

sales tax amount = sales tax rate of CP

=7.5% of CP

= 7.5/100 * $140

=$1050/100

=$10.5

total cost with tax = CP + tax rate

=$140 + $10.5

=$150.5

If a triangle has one interior angle measuring 102°, what is a possible pair of measurements for the other two
interior angles
it edmentum guided notes so it has to be a writing answer

Answers

Answer:

a2+a3=78

a2 and a3 must both be positive numbers that add up to 78

so a2 = a3 = 39

a1=102, a2=39, a3=39

Step-by-step explanation:

a1=102

a1+ a2 + a3=180 (all interrior angles of a triangle add up to 180)

a2+a3=78

a2 and a3 must both be positive numbers that add up to 78

Can someone check tell me why 1) is correct and why 2) is incorrect?!

Willing to give brainliest

Answers

Answer:

1) yes

2) no

Step-by-step explanation:

1)

[tex]\frac{1}{2} *\frac{2}{2}=\frac{2}{4}[/tex]

This is correct because 1 time 2 equals 2 and 2 times 2 equals 4.

2)

[tex]\frac{1}{2} * \frac{?}{?}= \frac{4}{6}[/tex]

[tex]\frac{1}{2} *\frac{4}{3} =\frac{4}{6}[/tex]

This is incorrect because the numerator multiplied is different from the denominator multiplied.

-------------------------------

[tex]\frac{1}{2}[/tex] =  [tex]\frac{2}{4}[/tex] = [tex]\frac{3}{6}[/tex] = [tex]\frac{4}{8}[/tex] = [tex]\frac{5}{10}[/tex] = [tex]\frac{6}{12}[/tex] ...

Sorry I didn't know how to explain that much but I hope you understand.

The histogram represents the distributions of essay scores for high school sophomores and juniors in a contest. Which statements are true about the data used to create the histogram? Select three options.

Answers

Question options:

A. The mean is the best comparison of the measures of center.

B. The juniors tended to have higher essay scores than the sophomores.

C. The medians of both data sets are equal.

D. The interquartile range is the best comparison of the measure of variability.

E. A histogram is the best way to show that both distributions are nearly symmetric.

Answer:

B. The juniors tended to have higher essay scores than the sophomores.

C. The medians of both data sets are equal.

E. A histogram is the best way to show that both distributions are nearly symmetric.

Explanation:

A histogram is used to represent data in a bar graph like manner. In a histogram, the bars lie along the x axis, each within the range of intervals while the y axis is used to count the frequency of occurrence that determines the area of each bar in the graph. Using a histogram, we can find B, C, and E from the options given above.

which of the following numbers must be added to complete the square in the equation below x^2+16x=5​

Answers

Answer:

8

Step-by-step explanation:

I was stumped on this question too so I figured I might as well learn how to do it. All you have to do is divide the middle number (16x) by 2. Pretty easy! Hope this helps anyone who needs it!

what is the value of w to the nearest degree

Answers

Answer hope it helps you

Determine whether the given triangles are congruent

Answers

Answer:

Yes they are congruent and it is explained below

Step-by-step explanation:

There are 5 postulates when testing for congruency and they are;

SSS, SAS, ASA, AAS and HL.

S represents side

A represents angle

H represents hypotenuse

L represents leg

Now, in this our question, we can see that the 3 angles and the three sides of both triangles are equal to each other.

Thus, we can make use of SSS postulate whereby we say the 3 sides of the first triangle are equal to the 3 sides of the second triangle.

Thus, they are congruent.

Help asappppp hbshhshshhshshsshshshs

Answers

9 on the top as well so 9 x 2 18 3x2 6 3

A factory puts 2,000 picture frames in each shipment. How many picture frames will the
factory put in 2 shipments?
picture frames

Answers

4,000

I know this because 2,000 • 2 = 4,000

in a class of 37 students,the number of students who like martial art only is double than the number of students who like athletics only .if students like both 4 like none of the games . find how many students like​

Answers

Answer:

number of students who like Athletics only = x = 10 students

number of students who like marshal arts only = 2x = 20 students

Step-by-step explanation:

In a class of 37 students, the number of students who like marshal arts only is

double than the number of students who like athletics only. If 3 students like both

and 4 like none of the games, find out how many students like:

i. Marshal arts

ii. Athletics​

Total students in the class = 37

number of students who like Athletics only = x

number of students who like marshal arts only = 2x

Number of students who like both = 3

Number of students who like none = 4

Total = athletics + marshal art + both + none

37 = x + 2x + 3 + 4

37 = 3x + 7

37 - 7 = 3x

30 = 3x

x = 30/3

x = 10

number of students who like Athletics only = x = 10 students

number of students who like marshal arts only = 2x

= 2(10)

= 20 students

A kite is staked to the ground and blowing in the wind. It currently has a string let out to a length of 45 meters. It has an altitude of 38 feet. To the nearest tenth of a degree, find the angle of elevation. ______°

Answers

Answer:

40.2

Step-by-step explanation:

To better analyze this problem, we can first draw a picture. Looking at the one attached, we can see that the stake is represented by the point in the bottom right of the screen, and the kite is 45 meters away from it, representing the 45. Then, the height is 38, and this is directly above the 45 meters away from the stake.

This forms a right triangle, and we can use sohcahtoa to help use. As we don't have the hypotenuse, we can use tan. Tan(θ) = opposite/adjacent, or 38/45. Thus, arctan(38/45) = θ, and plugging that into a calculator, we get 40.2 as our answer

The weight Wkg of a metal bar varies jointly as it's length L and the square of it's diameter D mm. If W=140 when D =4 and L=54, find D interm of W and L

Answers

Answer:

D = [tex]\sqrt{\frac{216W}{35L} }[/tex]

Step-by-step explanation:

From the given question, the expression showing the relationship among the weight, length and diameter of the metal bar is;

W [tex]\alpha[/tex] L[tex]D^{2}[/tex]

W = kL[tex]D^{2}[/tex]

where k is the constant of proportionality.

When W = 140, D = 4 and L = 54, then;

140 = k(54)[tex](4)^{2}[/tex]

      = 864k

k = [tex]\frac{140}{864}[/tex]

  = [tex]\frac{35}{216}[/tex]

k = [tex]\frac{35}{216}[/tex]

⇒ W = [tex]\frac{35LD^{2} }{216}[/tex]

So that;

35L[tex]D^{2}[/tex] = 216W

[tex]D^{2}[/tex] = [tex]\frac{216W}{35L}[/tex]

D = [tex]\sqrt{\frac{216W}{35L} }[/tex]

Jason and Donny painted a house and received $1.200. To complete the painting job Jason painted 4 hours 25 minutes and Donny spent 2 hours and 15 minutes. If they split the $1.200 in proportion to the amount of time each spent painting, how much did Donny receive?​

Answers

Answer:

$405

Step-by-step explanation:

Total hours = 4 hrs 25 min + 2 hrs 15 min = 6 hrs 40 min = 400 min

4 hrs 25 min = 265 minutes

2 hrs 15 min = 135 min

Donny's share = [tex]\frac{135}{400} of 1200[/tex]

                        [tex]=\frac{135}{400}*1200=135*3\\\\= 405[/tex]

Identify the least common multiple of:


(x + 1), (x - 1), & (x^2 - 1)

Answers

Given:

The expressions are [tex](x+1),\ (x-1)[/tex] and [tex](x^2-1)[/tex].

To find:

The least common multiple of given expressions.

Solution:

The expressions are [tex](x+1),\ (x-1)[/tex] and [tex](x^2-1)[/tex]. The factor forms of these expressions are:

[tex](x+1)=1\times (x+1)[/tex]

[tex](x-1)=1\times (x-1)[/tex]

[tex](x^2-1)=(x-1)(x+1)[/tex]                 [tex][\because a^2-b^2=(a-b)(a+b)][/tex]

The least common multiple is the product of all distinct factors with its highest degree. So,

[tex]L.C.M.=1\times (x+1)\times (x-1)[/tex]

[tex]L.C.M.=(x+1)(x-1)[/tex]

[tex]L.C.M.=x^2-1^2[/tex]                [tex][\because a^2-b^2=(a-b)(a+b)][/tex]

[tex]L.C.M.=x^2-1[/tex]

Therefore, the least common multiple of given expressions is [tex]x^2-1[/tex].

m∠RST =______because they are __________ (Fill in the blanks)

Answers

Answer:

∠RST = ∠SRU because they are alternate angles.

Step-by-step explanation:

Hope it helps .

Other Questions
what is the key code for a tamagotchi how does justice relate to fairness and equality? Translate the phrase into an algebraic expression.3 more than b Will mark brainliest PLEASE HEPWrite out a paragraph showing your understanding of scarcity what can be known about this situation? ASSIGNMENTSCOURSESAssignment - 1. Pre-TestAttempt 1 of 1SECTION 1 of 1QUESTICWorld War I marked the first war in which all of the following new technologies were used except foratomic bombsairplanestankspoison gas pls give me the answers On a coordinate plane, triangle A B C has points (negative 2, negative 1), (negative 2, negative 3), (negative 5, negative 1). Triangle ABC is translated 6 units to the right and 1 units up. What are the coordinates of C'? C'(4, 2) C'(4, 0) C'(4, 2) C'(1, 2) evaluate g(x)=x/x-3, if g(1/2) Look up lamentings in the dictionary. What did Lennox hear in the night? What is the human female reproductive cycle regulated by? 1) What is the control group? 2) What is the experimental group? 3) What is the independent (manipulated) variable? 4) What is the dependent (responding) variable? 5) What should Tina's conclusion be? Using the following image, complete the statement below. I got all the answers I need the ones that is blank A 21-year-old woman presents with a 3-month history of a black mole on her right calf. She tells you that the lesion is enlarging and expanding. It began to itch about 3 weeks ago, and it has bled 2 times. She thinks that there may have been a mole near the same spot previously, but she is not certain. Her general health is good; there is no history of chronic illness, hospitalizations, or surgeries. She works as a professional model for a large advertisement agency. She does not take any prescription medication; she does not use tobacco, alcohol, or recreational drugs. Although she has dark hair, she has a fair skin, and she says that she usually burns with even short sun exposure. She does occasionally use a tanning booth prior to modeling events and vacations. There is no family history of skin cancer.VS stable, she looks anxious, but she is otherwise well.There is a dark brown-black nodule on the right calf 1 cm in diameter.On the surface of the nodule, there is a tiny area of crusting. There are no hairs. The nodule is asymmetrical, and its border is sharply demarcated; the color is uniform, and the elevation is regular. There is a narrow (1-2 mm) rim of erythema around most of the nodule. She has a sprinkling (about 25-30 in all) of melanocytic nevi on her trunk and legs. There is no significant local or distal lymphadenopathy. The liver is not palpable. The remainder of the physical examination is unremarkable.What is the most likely diagnosis? Given the equation 4square root of x minus 3 = -12, solve for x and identify if it is an extraneous solution.A. x = 0, solution is extraneousB. x = 0, solution is not extraneousC. x = 12, solution is extraneousD. x = 12, solution is not extraneous Which expression is equivalent to 10k + 17 - 7j - 18 - 11k?-8jk - 1-7j - k - 1-7j + k + 1-8j - kPLEASE HURRY !!!!20 POINTS!!! The exponent on b when b^3 is multiplied by b^3 isA. 3B. 6C. 9 Hi Everyone hope u all r doin well Pls Answer What happens when dilute sulphuric acid is poured on a copper plate ? Indicate the equation of the given line in standard form, writing the answer in the equation box below.The line containing the longer diagonal of a quadrilateral whose vertices are A(2, 2), B(-2,-2), C(1, -1), and D16, 4).